Recent content by beefbrisket

  1. beefbrisket

    I Taking a partial trace of a multipartite state for measurement

    I am attempting to understand how POVMs fit in with quantum measurement, and I think I am getting tripped up in notation when it comes to multipartite systems. The situation is as follows: System: \rho_A Measurement instrument: \rho_B = |\phi\rangle\langle\phi| (pure state) The multipartite...
  2. beefbrisket

    Assumed value of pressure during quasistatic compression

    My text (Ian Ford - Statistical physics) describes an ideal gas system in a piston being quasistatically compressed by a piston head of area A under external force f. It assumes the system has a uniform pressure p. All good so far. Then it says: "the force pA equals the applied external force f"...
  3. beefbrisket

    I Computing inner products of spherical harmonics

    In this video, at around 37:10 he is explaining the orthogonality of spherical harmonics. I don't understand his explanation of the \sin \theta in the integrand when taking the inner product. As I interpret this integral, we are integrating these two spherical harmonics over the surface of a...
  4. beefbrisket

    I Sign mistake when computing integral with differential forms

    In one of Weintraub's intro texts on differential forms he introduces the "fundamental correspondence" between vectors and differential forms in R^3 (haven't been able to find any other sources using this name) and for the correspondence between vectors and 2-forms he indeed uses that ordering...
  5. beefbrisket

    I Sign mistake when computing integral with differential forms

    The question provides the vector field (xy, 2yz, 3zx) and asks me to confirm Stokes' theorem (the vector calc version) but I am trying to use the generalized differential forms version. So, I am trying to integrate \omega = xy\,dx + 2yz\,dy + 3zx\,dz along the following triangular boundary...
  6. beefbrisket

    I Meaning of cross terms in line element

    In a problem from Hartle's Gravity, we are asked to express the line element in non-Cartesian coordinates u, v which are defined with respect to x, y. I have no problem getting the new expression for the line element, but then we are asked if the new coordinate curves intersect at right angles...
Back
Top